Finance Test 3

Pataasin ang iyong marka sa homework at exams ngayon gamit ang Quizwiz!

The historical returns on large-company stocks, as reported by Ibbotson and Sinquefield and reported in your textbook, are based on the:

stocks of the 500 companies included in the S&P 500 index

Dan is a chemist for ABC, a major drug manufacturer. Dan cannot earn excess profits on ABC stock based on the knowledge he has related to his experiments if the financial markets are:

strong form efficient

The period 1926-2008 illustrates that U.S. Treasury bills:

can neither outperform or underperform inflation on an annual basis

Marine Expeditors has three divisions. Division A is the core of the business and represents 80 percent of the firm's operations. Division B is involved only with contractual short-term projects and therefore has about 8 percent less risk than Division A. Division C develops and markets new products and is about 12 percent riskier than Division A and about equal in size to Division B. The manager of Division A has suggested that the operations of his division be increased by 10 percent next year. The proposed project should probably be assigned a required return that is equal to _____ percent of the firm's weighted average cost of capital.

100

What was the average annual risk premium on small-company stocks for the period 1926-2008?

12.6 percent

What is the probability associated with a return that lies in the upper tail when the mean plus two standard deviations is graphed?

2.5 percent

a risky security has less risk than the overall market. What must the beta of this security be?

>0 but <1

Which one of the following statements is correct? Assume the pre-tax cost of debt is less than the cost of equity.

A firm may change its capital structure if the government changes its tax policies

Boone Brothers remodels homes and replaces windows. Ace Builders constructs new homes. If Boone Brothers considers expanding into new home construction, it should evaluate the expansion project using which one of the following as the required return for the project?

Ace Builder's cost of capital

The variance is the average squared difference between which of the following?

Actual return and average return

Based on the period 1926-2008, what rate of return should you expect to earn over the long-term if you are unwilling to bear risk?

Between 3 and 4 percent

Which one of the following combinations will always result in an increased dividend yield?

Decrease in the stock price combined with a higher dividend amount

Which one of the following statements concerning financial leverage is correct? (capital structure)

changes in the capital structure of a firm will generally change the firm's earnings per share

Which one of the following is defined as the average compound return earned per year over a multiyear period?

Geometric average return

Which of the following features are advantages of the dividend growth model? I. easy to understand II. model simplicity III. constant dividend growth rate IV. model's applicability to all common stocks

I and II only

The expected return on a security depends on which of the following? I. risk-free rate of return II. amount of the security's unique risk III. market rate of return IV. standard deviation of returns

I and III only

All else constant, which of the following will increase the aftertax cost of debt for a firm? I. increase in the yield to maturity of the firm's outstanding debt II. decrease in the yield to maturity of the firm's outstanding debt III. increase in the firm's tax rate IV. decrease in the firm's tax rate

I and IV only

Percentage returns: I. are easy to understand II. relay information about a security more easily than dollar returns do III. are not affected by the amount of the investment IV. can be easily separated into dividend yield and capital gain yield

I, II, III, and IV

Which of the following terms can be used to describe unsystematic risk? I. asset-specific risk II. diversifiable risk III. market risk IV. unique risk

I, II, and IV only

Based on the capital asset pricing model, investors are compensated based on which of the following? I. Market risk premium II. Portfolio standard deviation III. Portfolio beta IV. Risk-free rate

I, III, and IV only

The expected return on a security is currently based on a 22 percent chance of a 15 percent return given an economic boom and a 78 percent chance of a 12 percent return given a normal economcy. Which of the following changes will decrease the expected return on this security? I. an increase in the probability of an economic boom II. a decrease in the rate of return given a normal economy III. an increase in the probability of a normal economy IV. an increase in the rate of return given an economic boom

II and III only

The aftertax cost of which of the following are affected by a change in a firm's tax rate? I. preferred stock II. debt III. equity IV. capital

II and IV only

Which of the following are weaknesses of the dividend growth model? I. market risk premium fluctuations II. lack of dividends for some firms III. reliance on historical beta IV. sensitivity of model to dividend growth rate

II and IV only

Which of the following statements correctly relate to M&M Proposition I, with taxes? I. debt decreases the value of a firm II. the levered value of a firm exceeds the firm's unlevered value III. the weighted average cost of capital (WACC) is constant IV. the optimal capital structure is zero debt

II only

Which of the following will increase the value of a levered firm according to M&M Proposition I, with taxes? I. decrease in the amount of the debt II. increase in the value of the unlevered firm III. decrease in the tax rate IV. increase in the interest rate on the debt

II only

Which of the following will increase the cost fo equity for a firm with a beta of 1.1? I. decrease in the security's beta II. decrease in the market risk premium III. decrease in the risk-free rate IV> increase in the risk-free rate

III only

Kurt, who is a divisional manager, continually brags that his division's required return for its projects is 1 percent lower than the return required for any other division of the firm. Which one of the following most likely contributes the most to the lower rate requirement for Kurt's division?

Kurt's division is less risky than the other divisions

Which one of the following supports the theory that the value of a firm increases as the firm's level of debt increases?

M&M Proposition I, with taxes

Which one of the following states that a firm's cost of equity capital is a positive linear function of the firm's capital structure?

M&M Proposition II

Which one of the following is defined as a bell-shaped frequency distribution that is defined by its average and its standard deviation?

Normal distribution

Which one of the following best describes an arithmetic average return?

Return earned in an average year over a multiyear period

Investors requires a 4 percent return on risk-free investments. On a particular risky investment, investors require an excess return of 7 percent in addition to the risk-free rate of 4 percent. What is this excess return called?

Risk Premium

Which one of the following is most apt to cause a wise manager to increase a project's cost of capital?

She learns the project is riskier than previously believed.

Which one of the following catgeories has the widest frequency distribution of returns for the period 1926-2008?

Small-company stocks

Which one of the following is the positive square root of the variance?

Standard deviation

A firm has a return on equity of 12.4 percent according to the dividend growth model and a return of 18.7 percent according to the capital asset pricing model. The market rate of return is 13.5 percent. What rate should the firm use as the cost of equity when computing the firm's weighted average cost of capital (WACC)?

The arithmatic average of 12.4 percent and 18.7 percent

Which one of the following statements is correct concerning both the dollar return and the percentage return on a stock investment?

The dollar return is dependent on the size of the investment while the percentage return is not

Which one of the following statements is correct? (returns)

The higher the expected rate of return, the wider the distribution of returns

New Labs just announced that it has received a patent for a product that will eliminate all flu viruses. This news is totally unexpected and viewed as a major medical advancement. Which one of the following reactions to this announcement indicates the market for New Labs stock is efficient?

The price of New Labs stock increases rapidly ot a higher price and then remains at that price

Which one of the following statements matches M&M Proposition I?

The value of a firm is independent of the firm's capital structure

Which one of the following represents the present value of the interest tax shield?

TsubC * D

The rate of return on which one of the following is used as the risk-free rate?

U.S. Treasury bill

Which one of the following had the lowest standard deviation of returns for the period of 1926-2008?

U.S. Treasury bill

For the period 1926-2008, which one of the following had the smallest risk premium?

U.S. Treasury bills

Which one of the following statements in true regarding the period 1926-2008?

U.S. Treasury bills had a positive average real rate of return

The historical record for the period 1926-2008 shows that the annual nominal rate of return on:

U.S. Treasury bills have has a positive rate of return for every year in the period

Which one of the following is minimized when the value of a firm is maximized?

WACC

Which one of the following conditions exists at the point where a firm maximizes its value?

WACC is minimized

Which one of the following statements is correct, all else held constant?

a decrease in a firm's WACC will increase the attractiveness of the firm's incestment options

A stock is expected to return 13 percent in an economic boom, 10 percent in a normal economy, and 3 percent in a recessionary economy. Which one of the following will lower the overall expected rate of return on this stock?

a decrease in the probability of an economic boom

Based on the capital asset pricing model, which one of the following must increase the expected return on an individual security, all else constant?

a decrease in the risk-free rate given a security beta of 1.06

According to the Efficient Markets Hypothesis, professional investors will earn:

a dollar return equal to the value paid for an investment

Which one of the following statements regarding bankruptcy filing is correct?

a firm can file for Chapter 11 bankruptcy even if the firm is solvent

Assume both corporate taxes and financial distress costs apply to a firm. Given this, the static theory of capital structure illustrates that:

a firm's value and its weighted average cost of capital are inveresly related

Which one of the following statements related to the static theory of capital structure is correct?

a firm's value is maximized when a firm operates at its optimal debt level

Which one of the following best defines legal bankruptcy?

a legal proceeding for liquidating or reorganizing a business

Which one of the following portfolios will have a beta of zero?

a portfolio comprised solely of U.S. Treasury bills

Which one of the following statements is accurate for a levered firm?

a reduction in the risk level of a firm will tend to decrease the firm's WACC

Which one of the following statements related to the security market line is correct?

a security with a beta of 1.54 will plot on the security market line if it is correctly priced

Derek's is a brick-and-mortar toy store. The firm is considering expanding its operations to include Internet sales. Which one of the following would be the best firm to use in a pure play approach to analyzing this proposed expansion?

a toy store that only sells online

Which one of the following is the best example of unsystematic risk?

a warehouse fire

In the process of liquidation, some types of claims receive preference over other claims. Which one of the following determines which type of claim is paid first?

absolute priority rule

Semi-strong form market efficiency states that the value of a security is based on:

all publicly available information

Which one of the following statements is correct? (security and security market line)

an underpriced security will plot above the security market line

A firm has multiple divisions of similar nature, yet varying degrees of risk. Which one of the following would be the most appropriate, yet relatively easy, means of assigning discount rates to each of its proposed investments?

assign every project a rate equal to the firm's WACC plus or minus a subjective adjustment

Old Town Industries has three divisions. Division X has been in existence the longest and has the most stable sales. Division Y has been in existence for five years and is slightly less risky than the overall firm. Division Z is the research and development side of the business. When allocating funds, the firm should probably:

assign the highest cost of capital to division Z becasue it is most likley the riskiest of the three divisions

A firm uses its weighted average cost of capital to evaluate the proposed projects for all of its varying divisions. By doing so, the firm:

automatically gives preferential treatment in the allocation of funds to its riskiest division

An efficient capital market is best defined as a market in which security prices reflect which one of the following?

available information

Which one of the following will generally receive the highest priority in a bankruptcy liquidation, assuming the absolute priority rule is followed?

bankruptcy administrative expenses

Which of the following statements regarding bankruptcy is correct?

bankruptcy courts have "cram-down" powers

When using the pure play approach for a proposed investment, a firm is primarily seeking a rate of return that:

best matches the risk level of the proposed investment

Which one of the following measures the amount of systematic risk present in a particular risky asset relative to that in an average risky asset?

beta coefficient

Which one of the following is the computation of the risk premium for an individual security? E(r) is the expected return on the security, rsubf is the risk-free rate, beta is the security's beta, and E(r)subm is the expected rate of return on the market.

beta*[E(r)subm - rsubf]

Which one of the following is the equity risk arising from the daily operations of a firm?

business risk

The capital asset pricing model:

considers the time value of money

Which one of the following is the minimum required rate of return on a new investment that makes that investment attractive?

cost of capital

Lester lent money to The Corner Store by purchasing bonds issued by the store. The rate of return that he and the other lenders require is referred to as the:

cost of debt

Katie owns 100 shares of ABC stock. Which one of the following terms is used to refer to the return that Katie and the other shareholders require on their investment in ABC?

cost of equity

The weighted average cost of capital is defined as the weighted average of a firm's:

cost of equity and its aftertax cost of debt

M&M Proposition II, without taxes, states that the:

cost of equity increases as a firm increases its debt-equity ratio

The level of financial risk to which a firm is exposed is dependent on the firm's:

debt-equity ratio

Which one of the following is the best example of systematic risk?

decrease in gross domestic product

Which one of the following terms best refers to the practice of investing in a variety of diverse assets as a means of reducing risk?

diversification

Which one of the following is the hypothesis that securities markets are efficient?

efficient markets hypothesis

Mary owns a risky stock and anticipates earning 16.5 percent on her investment in that stock. Which one of the following best describes the 16.5 percent rate?

expected return

The security market line is a linear function which is graphed by plotting data points based on the relationship between which two of the following variables?

expected return and beta

The security market line is defined as a positively sloped straight line that displays the relationship between which two of the following variables?

expected return and beta

Which one of the following terms is inclusive of both direct and indirect bankruptcy costs?

financial distress costs

Which one of the following statements concerning financial leverage is correct? (financial leverage)

financial leverage magnifies both profits and losses

Which one of the following is the equity risk arising from the capital structure selected by a firm?

financial risk

You need to use the pure play approach to assign a cost of capital to a proposed investment. Which one of the following characteristics should you most concentrate on as you search for an appropriate pure play firm?

firm operations

Which one of the following best describes a portfolio

gorup of assets held by an investor

The use of borrowing by an individual to adjust his or her overall exposure to financial leverage is referred to as:

homemade leverage

The cost of capital for a project depends primarily on which one of the following?

how the project uses its funds

Which one of the following statements is correct? (risky security)

if a risky security is correctly priced, its expected risk premium will be positive

Which one of the following is an example of systematic risk?

increase in consumption created by a reduction in personal tax rates

Which one of the following will decrease the aftertax cost of debt for a firm?

increase in tax rates

Which one of the following will increase the cost of equity, all else held constant?

increase in the dividend growth rate

World United stock currently plots on the security market line and has a beta of 1.04. Which one of the following will increase that stock's rate of return without affecting the risk level of the stock, all else constant?

increase in the market risk-to-reward ratio

Which one of the following will affect the capital structure weights used to compute a firm's weighted average cost of capital?

increase in the market value of the firm's common stock

A firm that uses its weighted average cost of capital as the required return for all of its investments will:

increase the risk level of the firm over time

In an efficient market, the cost of equity for a risky firm does which one of the following according to the security market line?

increases in direct relation to the stock's systematic risk

a firm has a cost of equity of 13 percent, a cost of preferred of 11 percent, and an aftertax cost of debt of 6 percent. Given this, which one of the following will increase the firm's weighted average cost of capital?

increasing the firm's beta

Which one of the following is an example of a direct bankruptcy cost?

incurring legal fees for the preparation of bankruptcy filings

Which one of the following terms applies to the costs incurred by a firm which is trying to avoid filing for bankruptcy?

indirect bankruptcy costs

Paying interest reduces the taxes owned by a firm. Which one of the following terms applies to this relationship?

interest tax shield

Which one of the following has the narrowest distribution of returns for the period 1926-2008?

intermediate-terms government bonds

Julie wants to create a $5,000 portfolio. She also wants to invest as much as possible in a high risk stock with the hope of earning a high rate of return. However, she wants her portfolio to have no more risk than the overall market. Which one of the following portfolios is most apt to meet all of her objectives?

invest $2,500 in a risk-free asset and $2,500 in a stock with a beta of 2.0

The cost of preferred stock:

is equal to the stock's dividend yield.

The static theory of capital structure assumes a firm:

is fixed in terms of its assets

a prepack:

is the joint filing of both a bankruptcy filing and a creditor-approved reorganization plan

Which one of the following is a direct bankruptcy cost?

legal and accounting fees related to a bankruptcy proceeding

Which one of the following is the primary determinant of an investment's cost of capital?

level of risk

Which one of the following terms refers to the termination of a firm as a going concern?

liquidation

Over the period of 1926-2008 (business):

long-term government bonds underperformed long-term corporate bonds

The lower the standard deviation of returns on a securty, the _____ the expected rate of return and the _____ the risk.

lower; lower

The systematic risk principle states that the expected return on a risky asset depends only on which one of the following?

market risk

Which one of the following is the slope of the security market line?

market risk premium

The addition of a risky security to a fully diversified portfolio:

may or may not affect the portfolio beta

systematic risk is:

measured by beta

If a security plots to the right and below the security market line, then the security has ____ systematic risk than the market and is ____.

more; overpriced

When, if ever, will the geometric average return exceed the arithmetic average return for a given set of returns?

never

Assume you own a portfolio of diverse securities which are each correctly priced. Given this, the reward-to-risk ratio:

of each security must equal the slope of the security market line

If the financial markets are semi-strong form efficient, then:

only individuals with private information have a marketplace advantage

Stock A comprises 28 percent of Susan's portfolio. Which one of the following terms applies to the 28 percent?

portfolio weight

Farmer's Supply, Inc. is considering opening a clothing store, which would be a new line of business for the firm. Management has decided to use the cost of capital of a similar clothing store as the discount rate that should be used to evaluate this proposed expansion. Which one of the following terms is used to describe the approach Farmer's Supply is taking to establish an appropriate discount rate for the project?

pure play approach

Peterboro recently defaulted on a bank loan. To avoid a bankruptcy proceeding, the bank agreed to a composition. This composition would do which one of the following?

reduce the amount of the loan payments so Peterboro can pay timely

Diversifying a portfolio across various sectors and industries might do more than one of the following. However, this diversification must do which one of the following?

reduce the portfolio's unique risks

Greenwood Motels has filed a petition for bankruptcy but hopes to continue its operations both during and after the bankruptcy process. Which one of the following terms best applies to this situation?

reorganization

Which one of the following is a key provision of the Bankruptcy Abuse Prevention and Consumer Protection Act of 2005?

right granted to creditors to file their own reorganization plan once a firm is in bankruptcy for 18 months

Kelly's uses the firm's weighted average cost of capital (WACC) as the required return for some of its projects. For other projects, the firms uses a rate equal to WACC plus 1 percent, while another set of projects is assigned rates equal to WACC minus some amount. Which one of the following factors should be the key factor the firm uses to determine the amount of the adjustment it will make when assigning the project a discount rate?

risk level of project

Ted is trying to decide what cost of capital he should assign to a project. Which one of the following should be his primary consideration in this decision?

risk level of the project

Which one of the following describes systemic risk?

risk that affects a large number of assets

Which one of the following is the vertical intercept of the security market line?

risk-free rate

T.L.C. Enterprises just revised its capital structure from a debt-equity ratio of .30 to a debt-equity ratio of .45. The firm's shareholders who prefer the old capital structure should:

sell some shares and loan out the sale proceeds

Over the period of 1926-2008, which one of the following investment classes had the highest volatility of returns?

small-comany stocks

Which one of the following is the most apt to have the largest risk premium in the future based on the historical record for 1926-2008?

small-company stocks

Which one of the following is the best example of an announcement that is most apt to result in an unexpected return?

statement by a firm that it has just discovered a manufacturing defect and is recalling its product

Which one of the following is the theory that a firm should borrow up to the point where the additional tax benefit from an extra dollar of debt equals the additional costs associated with financial distress from that additional debt?

static theory of capital structure

Which one of the following could cause the total return on an investment to be a negative rate?

stock price that declines over the investment period

If the financial markets are efficient then:

stock prices should only respond to unexpected news and events

Kate is the CFO of a major firm and has the job of assigning discount rates to each project under consideration. Kate's method of doing this is to assign an incrementally higher rate as the risk level of the project increases over that of the current firm. Likewise, she asigns lower rates as the risk level declines. Which one of the following approached is Kate using to assign the discount rates?

subjective approach

The computation of which one of the following requires assigning every proposed investment to a particular risk class?

subjective cost of capital

One year ago, you purchased 100 shares of a stock. This morning you sold those shares and realized a total return of 8.2 percent. Given this information, you know for sure the:

sum of the dividend yield and the capital gains yield is 8.2 percent

The risk premium for an individual security is based on which one of the following types of risk?

systematic

Which one of the following statements is correct related to the dividend growth model approach to computing the cost of equity?

the annual dividend used in the computation must be for year one if you are using today's stock price to compute the return

Which one of the following statements is the core principle of M&M Proposition 1, without taxes?

the capital structure of a firm is totally irrelevant

Which one of the following statements is correct? (stock and tax rate)

the cost of preferred stock is unaffected by the issuer's tax rate

Which one of the following is correct based on the static theory of capital structure?

the costs of financial distress decrease the value of a firm

All else held constant, the weighted average cost of capital for a risky, levered firm will decrease if:

the firm's bonds start selling at a premium rather than at a discount

The beta of a risky portfolio cannot be less than _____ nor greater than ____.

the lowest individual beta in the portfolio; the highest individual beta in the portfolio

Black Stone Furnaces wants to build a new facility. The cost of capital for this investment is primarily dependent on which one of the following?

the nature of the investment

Consider a portfolio comprised of four risky securities. Assume the economy has three states with varying probabilities of occurrence. Which one of the following will guarantee that the portfolio variance will equal zero?

the portfolio expected rate of return must be the same for each economic state

Assume the securities markets are strong-form efficient. Given this assumption, you should expect which one of the following to occur?

the price of each security in that market will frequently fluctuate

The average risk premium on long-term government bonds for the period 1926-2014 was equal to:

the rate of return on the bonds minus the T-bill rate

Which one of the following statements is correct concerning capital structure weights?

the repurchase of preferred stock will increase the weight of debt

Which one of the following is an implication of M&M Proposition II, without taxes?

the risk of equity depends on both the degree of financial leverage and the riskiness of the firm's operations

Over the period of 1926-2008 (bonds):

the risk premium on stocks exceeded the risk premium on bonds

You are assigned the task of computing the expected return on a portfolio containing several individual stocks. Which one of the following statements is correct concerning this task?

the summation of the return deviation from the portfolio expected return for each economic state must equal zero

Assume you are comparing two firms that are identical in every aspect, except one is levered and one is unlevered. Which one of the following statements is correct regarding these two firms?

the unlevered firm will have higher EPS at relatively low levels of EBIT

The expected rate of return on Delaware Shores, Inc. stock is based on three possible states of the economy. These states are boom, normal, and recession which have probabilities of occurence of 20 percent, 75 percent, and 5 percent, respectively. Which one of the following statements is correct concerning the variance of the returns on this stock?

the variance must be positive provided that each state of the economcy produces a different expected rate of return

Standard deviation measures _____ risk while beta measures _____ risk.

total; systematic

Which one of the following best exemplifies unsystematic risk?

unexpected increase in the variable costs for a firm

Portfolio diversification eliminates which one of the following?

unsystematic risk

The standard deviation measures the _____ of a security's returns over time.

volatility

Which one of the following represents the rate of return a firm must earn on its assets if it is to maintain the current value of its securities?

weighted average cost of capital

Which one of the following i sthe pre-tax cost of debt?

weighted average yield-to-maturity on the firm's outstanding debt

When is a firm insolvent from an accounting perspective?

when the firm has a nergative net worth

You are comparing two possible capital structures for a firm. The first option is an all-equity firm. The second option involved the use of $3.8 million of debt. The break-even point between these two financing options occurs when the earnings before interest and taxes (EBIT) are $428,000. Given this, you know that leverage is beneficial to the firm:

whenever EBIT exceeds $428,000

A portfolio is comprised of 35 securities with varying betas. The lowest beta for an individual security is .74 and the highest of the security betas of 1.51. Given this information, you know that the portfolio beta:

will be greater than or equal to 0.74 but less than or equal to 1.51

All else constant, an increase in a firm's cost of debt:

will result in an increase in the firm's cost of capital

Which one of the following represents the amount of compensation an investor should expect to receive for accepting the unsystematic risk associated with an individual security?

zero


Kaugnay na mga set ng pag-aaral

Chapter 3 The Cellular Level of Organization

View Set

Chapter 10: Antimicrobial Treatment

View Set

Financial Industry Regulatory Authority (FINRA)

View Set

Chapter 9 - the physical basis of memory formation

View Set

accounting 320 Intangibles chapter 12

View Set